0
$\begingroup$

On page 12 of the paper, there is a formula about super Poisson bracket on a Lie super group $G$: \begin{align} \{\phi, \psi\} = \sum_{\mu, \nu} (-1)^{|\phi||\nu|} r^{\mu \nu} ( R_{\mu} \phi R_{\nu} \psi - L_{\mu} \phi L_{\nu} \psi ), \end{align} where $\phi, \psi$ are functions on $G$.

I am trying to verify that the bracket satisfies \begin{align} \{\psi, \phi\} = - (-1)^{|\psi| |\phi|} \{\phi, \psi\}. \end{align}

We have \begin{align} \{\psi, \phi\} & = \sum_{\mu, \nu} (-1)^{|\psi||\nu|} r^{\mu \nu} ( R_{\mu} \psi R_{\nu} \phi - L_{\mu} \psi L_{\nu} \phi ) \\ & = \sum_{\mu, \nu} (-1)^{|\psi||\mu|} r^{\nu \mu} ( R_{\nu} \psi R_{\mu} \phi - L_{\nu} \psi L_{\mu} \phi ) \\ & = \sum_{\mu, \nu} (-1)^{|\psi||\mu|} r^{\nu \mu} ( -(-1)^{|R_{\nu}\psi | | R_{\mu} \phi|} R_{\mu} \phi R_{\nu} \psi + (-1)^{|L_{\nu} \psi| |L_{\mu} \phi|} L_{\mu} \phi L_{\nu} \psi ) \\ & = \sum_{\mu, \nu} (-1)^{|\psi||\mu|} (-1)^{|\mu||\nu|} r^{ \mu \nu} ( (-1)^{|R_{\nu}\psi | | R_{\mu} \phi|} R_{\mu} \phi R_{\nu} \psi - (-1)^{|L_{\nu} \psi| |L_{\mu} \phi|} L_{\mu} \phi L_{\nu} \psi ). \end{align} Therefore \begin{align} & (-1)^{|\psi||\phi|}\{\psi, \phi\} \\ & = \sum_{\mu, \nu} (-1)^{|\psi||\phi|} (-1)^{|\psi||\mu|} (-1)^{|\mu||\nu|} r^{ \mu \nu} ( (-1)^{|R_{\nu }\psi | | R_{\mu} \phi|} R_{\mu} \phi R_{\nu} \psi - (-1)^{|L_{\nu} \psi| |L_{\mu} \phi|} L_{\mu} \phi L_{\nu} \psi ). \end{align} Therefore we need to show that \begin{align} & (-1)^{|\psi||\phi| + |\psi||\mu| + |\mu||\nu| + |R_{\nu }\psi | | R_{\mu} \phi| + |\phi| |\nu| } = 1, \\ & (-1)^{|\psi||\phi| + |\psi||\mu| + |\mu||\nu| + |L_{\nu }\psi | | L_{\mu} \phi| + |\phi| |\nu| } = 1. \end{align} In general, do we have the above identities? How to prove them? Thank you very much.

$\endgroup$
2
  • $\begingroup$ Since this problem was resolved in the comments to my answer for your previous question mathoverflow.net/questions/261506/… , I suggest that maybe this question should be removed. $\endgroup$ Feb 7, 2017 at 16:47
  • $\begingroup$ @VladimirDotsenko, thank you very much for your suggestions. Maybe let me keep this question because I reference to this question in the comments of my previous question. $\endgroup$ Feb 7, 2017 at 17:09

1 Answer 1

0
$\begingroup$

This question was solved by Vladimir Dotsenko in the comments of the question.

$\endgroup$

Your Answer

By clicking “Post Your Answer”, you agree to our terms of service and acknowledge you have read our privacy policy.

Not the answer you're looking for? Browse other questions tagged or ask your own question.